Verify an inequality using induction: $\frac1{2n}\le \frac{1\cdot3\cdot5\cdots(2n-1)}{2\cdot4\cdot6\cdots(2n)}$

discrete mathematicsinductionproof-explanationsolution-verification

How can I verify this inequality by using induction?

$$\frac1{2n}\le \frac{1\cdot3\cdot5\cdots(2n-1)}{2\cdot4\cdot6\cdots(2n)}$$

I understand the principle of induction, but I'm struggling with how my teacher is solving this problem. I Highlighted the area I'm stuck at In the solution – why are we able to take $1/2k$ and see if its less than the left side of our inducted hypothesis? (view the highlighted part of the photo)

Any suggestions would be great – I really don't understand how to solve this problem. If I could get a step by step walk through of this proof with side notes, I'd be very grateful

enter image description here

Best Answer

In this screenshot from your image call the LHS and RHS of the first inequality (which is a consequence of the induction hypothesis) $B$ and $C$ respectively. The first inequality is stating that $\boxed{B\le C}$.

Call the LHS of the second inequality, marked in yellow by you, to be $A$.

You want to prove $A\le C$, and you have already proved $B\le C$ (mentioned in the first block above), so
if you can prove $A\le B$, then you can combine the consequence of the first inequality, i.e $\boxed{B\le C}$ with $A\le B$ to get $$A\le B \le C \implies A \le C$$ which would prove the claim. So we want to prove (as written by your teacher) that $A\le B$.

enter image description here

Related Question